What is Rod: Definition and 1000 Discussions

Röd (Swedish for Red) is the eighth studio album by Swedish alternative rock band Kent. It was released as digital download exclusively through the band's website on 5 November 2009 and physically on 6 November 2009. The first single from the album, "Töntarna", was released as digital download on 5 October 2009.
Röd is available in a standard edition and a deluxe edition box. The deluxe edition box version features the 11-track CD, a USB flash drive with high quality MP3 files as well as AIFF files, three 10" records which between them contain the whole album, and a 118-page book containing lyrics, abstract pictures and photographs. Due to distribution difficulties the deluxe edition was delayed until 11 November 2009.

View More On Wikipedia.org
  1. V

    Hinged rod rotating, falling and hitting a mass

    Assuming no friction anywhere, no drag and perfect inelastic collision Using conservation of mechanical energy i can determine the rotational speed of the rod right before collision occurs. mgh=1/2*i*w^2 center of mass falls 1/2*L so we have: M*g*1/2*L = 1/2*(1/3*M*L^2)*w^2 Solving for w...
  2. N

    Engineering Find the velocity and angular velocity of the rod and point

    Hi everyone :)! I resolve this problem with components method and trigonometry method. My results with components method its okay, but i can´t obtain the correct VE velocity. Im sure that the problem its in the angles, but i don't know how to fix it. The correct answers: -Angular velocity...
  3. Ugnius

    Thin Charged Isolated Rod -- Find the electric field at this point

    Hi , I've been trying to manage a solution in my head and i think I'm on the right path , i just need some approval and maybe some tips. So it's obvious I can't solve this without integration because law's only apply to point charges , and i can't shrink this object to a point as i could do with...
  4. S

    Force exerted by rod on a mass moving in vertical circle

    (A) and (B) are obviously wrong but I think both (C) and (D) are correct. At the top, the forces acting on the mass are tension and weight, both directed downwards so the equation of motion will be: $$\text{Tension}+\text{Weight}=m.a$$ $$\text{Tension}=m.a-\text{Weight}$$ Based on that...
  5. K

    Electric field in a rotating rod in a magnetic field

    The first part of the problem seems easy enough, the free electrons in the wire would move in a circle owing to an electric field that would be induced in the rod which would provide the centripetal force for the same (Please correct me if I am wrong). So we have $$eE=mω^2x$$, where e is the...
  6. L

    Tangential velocity of rotating rod

    1) ##LT\sin(\frac{\pi}{2}-\theta)-\frac{L}{2}mg\sin\theta=0\Rightarrow T=\frac{mg}{2}\tan\theta##. ##N_{x}-T=0, N_{y}-mg=0\Rightarrow N=\sqrt{N_x ^2+N_y ^2}=mg\sqrt{(\frac{\tan\theta}{2})^2 +1}## 2) ##E_{k_{fin}}=mg\frac{L}{2}[1+\cos\theta]## 3)...
  7. V

    Reason for glass rod losing electrons

    I am not sure if the explanation below is enough. This is a high school level question. When rubbing occurs between glass and silk, then heat energy is produced which provides the energy needed to free up electrons in outermost orbits of atoms in silk or glass. But silk has very tightly bound...
  8. M

    Engineering Equation of motion for the translation of a single rod

    Hello, Given the statement a described above. To find the forces at point D I drawn a kinematic scheme and FBD of rod CD. But why am I allowed to ignore the mass of 50 kg, the forces at point B and point A? I know the are some rules about this, but I just can't remember them anymore.. The...
  9. J

    Understanding the Interaction Between a Charged Rod and a Metal Sphere

    (A) incorrect, because opposite signs attract, and the sphere would've been drawn to the charged rod. (B) correct, according to the answer key, but if the charge of the sphere and the charge of the rod are the same, then wouldn't they repel each other? I'm confused as to why this is the correct...
  10. P

    Lagrangian mechanics - rotating rod

    Hello, It might sound silly, but when I try to calculate the kinetic energy of a rotating rod to form the Langrangian (and in general), why it has both translational and rotational kinetic energy? Is it because when I consider the moment of Inertia about the centre I need to include the...
  11. B

    What are the forces exerted by a wall on a hinged rod?

    I have a dilemma. If I look at the diagram and say the sum of the forces in x and y direction has to be zero, then I will simply conclude that the force of a wall on a revolvable rod is the blue N as drawn. But what if the force is actually the green N? To me, it makes more sense because if...
  12. S

    I Relativity's Effect on Long Rod Shape and Speed

    Hello All I pivot a long rigid rod at one quarter its length and gradually accelerate the tip of the short end to close to the speed of light. An observer is standing some distance away from the mechanism, so that he/she can see the whole length of the rod. What would the shape of the rod...
  13. greg_rack

    Angular velocity of a weighted rod left free to rotate around a pivot

    Hi guys, I don't really know how to solve this problem. The point is finding ##\omega## when ##m_2## passes from ##m_1##'s original position. Ideally, I'm thinking about some conservation of energy/momentum to apply here, but I'm quite confused. Any hint?
  14. L

    MHB 4 rod Tower of Hanoi proof

    I am having trouble solving part 2, for $ W_{\frac{n(n+1)}{2}} \leq 2^{n} (n-1) + 1 , n \geq 0 $ I know that $W_{m} \leq 2*W_{m-k} + 2^{k} – 1, 0 \leq k \leq m$ Let $m = \frac{n(n+1)}{2}$ So now $W_{\frac{n(n+1)}{2}} \leq 2*W_{\frac{n(n+1)}{2} - k} + 2^{k} - 1, 0 \leq k \leq...
  15. Kaguro

    Degrees of freedom with a particle and a rod

    The rod itself should have 3 translational+2 rotational DOF. The particle on top of the rod has one additional DOF. So total should be 6. But answer given is 4. What I'm thinking wrong?
  16. I

    Electric Field from a Charged Semicircular Rod

    Charge QQ is uniformly distributed along a thin, flexible rod of length LL. The rod is then bent into the semicircle shown in the figure (Figure 1).Find an expression for the electric field E⃗ E→ at the center of the semicircle. Hint: A small piece of arc length ΔsΔs spans a small angle...
  17. A

    Charges, rod and magnetic field

    I have some difficulties in solving this problem. This is what I did. I wrote down the equation of motion for the masses. For the first point \begin{equation} m\ddot{\textbf{r}}_1=\textbf{F}_1=q\dot{\bar{\textbf{r}}}_1\times...
  18. S

    Angular speed of rod shot by bullet

    1) Applying conservation of linear momentum: $$m.u = M.V + m.v$$ where ##V## is final linear speed of the rod $$V=\frac{m.u-m.v}{M}$$2) Applying formula of circular motion: $$V=\omega . r$$ $$\omega = \frac{\left(\frac{mu-mv}{M} \right)}{\frac{1}{2}L-x}$$ Is this correct?And can this be...
  19. C

    Density of balls submerged in a liquid and connected by a hard rod

    Performing force balance on the two balls, I obtain ## T+\frac{4\pi}{3}g \rho_3 R_1 ^3 = \frac{4\pi}{3}g \rho_1 R_1^3 ## ## T+\frac{4\pi}{3}g \rho_2 R_2^3 = \frac{4\pi}{3}g \rho_3 R_2^3 ## from which I obtain ## \rho_3 = \frac{\rho_1 R_1^3 + \rho_2R_2^3 }{R_1^3 + R_2 ^3 }## and ## 2T...
  20. R

    Rotation of a rod fastened to a wire

    Hi, I started with calculating the moment of inetria of the rod: I = ⅓ML^2 + M(3/2 * L)^2 = 31/12 ML^2 and I thought that the reaction force in the first case will be equal to centrifugal force: F1 = Mω^2*(3/2)L Angular velocity is calculated from the conservation of energy: Mg3/2*L=1/2 * Iω^2...
  21. Y

    Effects of notch's position along a rod that is in tension

    Say I have a rod with a notch at the very centre and another ro with a notch at the very end. Both rods are identical in length radius materials etc. Who would be impacted by the notch more? And if then both were to support a load at the end of the rod (both in tension), what effects would the...
  22. Hamiltonian

    Finding the net elongation in a rod due to its own weight

    I realized that the tension in the rod is not uniform and found it to be ##T = Wx/L## I found this by splitting the rod into two sections one of length ##x## and the other of length ##L-X## where x is the length from the base of the hanging rod To find the total elongation in the rod I...
  23. S

    Normal force on a rod by the rim of a bowl

    I want to ask the direction of normal force acting on the rod by the rim of the bowl. Is the direction perpendicular to the rod or will it be directed horizontally to the left? My guess would be horizontally to the left because the normal force would be perpendicular to the "plane / surface"...
  24. S

    Force at the end of a Rod at rest on an inclined surface

    Question diagram, attempt at solution below I need to cancel some of the terms in the moment equation but a not sure which ones to start with. I don’t know μ so can not calculate FA, so should probably substitute FA = RB2.
  25. sahilmm15

    A problem regarding static charge -- rubbing a metal rod with wool

    A metal rod held in hand and rubbed with wool will not show any sign of of being charged. However, if a metal rod with a wooden or plastic handle is rubbed without touching its metal part, it shows signs of charging. Why??
  26. greg_rack

    Period of a metal rod oscillating in a magnetic field

    This problem honestly got me in big confusion. I managed to find the angle ##\theta## at which the rod rests by equalling the components of weight and Lorentz's force... but from this point on I really don't know how to manage the harmonic oscillation part.
  27. Lord Crc

    B What happens when you push a rod?

    Note, I put this question here because I imagined it fit the best here, but mods feel free to move it if there's a better place. So my girlfriend asked me a question today I couldn't answer. Imagine you have a crystal rod (for simplicity), and you push it a bit at one end, in the axial...
  28. S

    Find the angular acceleration of two reflectors attached to a rod

    Okay so I actually have the answer because my teacher basically just gave it to me, but I would really like to know why I was even wrong in the first place. Here's my steps: 1. Knowing the momentum transfer per unit area is described by: 1/A dp/dt = S/c. I can begin by relating some known...
  29. mingyz0403

    Engineering Motion of two balls connected by a rod

    Since there is no friction, there is no radial force acting on Ball B after the pin is remove. Therefore the radial acceleration of Ball B is zero. I don't know how to determine the transverse components of the acceleration of Ball B. I looked at the textbook solution. It takes moment about the...
  30. mingyz0403

    Engineering Newton’s Second Law of Motion — Collar sliding on a rotating rod

    The soultion used polar corrdinates. Acceleration in polar corrdinates have radial and transeverse components.When calculating the acceleration of collar respect to the rod, the solution only calculates the radial component of acceleration. Is it because the collar is on the rod, so the...
  31. Spinnor

    Misc. Bending a spring steel rod to shape and heat treat, DIY?

    There used to be sold a style bicycle handlebar bags that used what I think is a formed spring steel rod that fit over the handlebars and looped under the handlebar stem that supported a handlebar bag. For whatever reason this style does not appear to be available any more. I think it is a...
  32. D

    Why is the result different in Method 2 for the rotating rod experiment?

    Method 1: Simply conserving angular momentum about the the fixed vertical axis and conserving energy gives ##v=3##, which is correct according to my book. Method 2: Conserving angular momentum when the two rings reach distance ##x## from the centre gives ##(0.01+2x^2) \omega =0.9## Also in the...
  33. archaic

    Potential due to a rod with a nonuniform charge density

    I'm not sure I understand why I need to use ##d##.. Maybe they want me to have the potential be zero at ##A##? In any case, I have found$$V(B)=\alpha k\int_0^L\frac{x}{\sqrt{b^2+\left(x-\frac{L}{2}\right)^2}}dx+C=\frac{\alpha...
  34. K

    Kinematics of a Pendulum with Two Different Masses

    Summary:: Classical problem about a pendulum! The problem itself: My FBD: I want to solve the problem with vectors, I think that you can use energy principle somehow. If we define the vector ##\vec{O}_B=\begin{bmatrix}0\\ -1\end{bmatrix}## and define a rotational matrix where...
  35. A

    Ball hits a pivoting rod, what torque changes the angular momentum of the ball?

    Okay, i know that as a ball collides with a pivoting rod on an axis, the ball has angular momentum. Therefore after the collision, the ball is stopped or slowed, and the rod swings. The ball provides a force and torque to the rod. But if I isolate the ball, isn't the only thing acting on the...
  36. Data Base Erased

    Why Does a Moving Rod Appear Inclined in Different Reference Frames?

    Ateempt of solution: There are two key coordinates in this scenario, the leftmost tip of the rod, which in ##S'## is ##C_{0} = (t', 0, ut',0)## and the opposite tip ##C_{1} = (t', L,ut',0)## An angle ##\phi## could be found through a relationship such as ##tan(\phi) = \frac{ \Delta x}{ \Delta...
  37. Hamiltonian

    A rotating rod acted upon by a perpendicular force

    $$\tau = I\alpha$$ $$FL/2 = I\omega^2L/2$$ $$T = 1/\theta \sqrt{F/I}$$ would this be correct? I came up with this more basic question to solve a slightly harder question so I do not know the answer to the above-stated problem.
  38. D

    Angular speed of a falling rod

    I have seen the solution and understand it. The solution defines θ to be the angle between the falling rod and the table. It then equates initial PE to the final (PE+KE) where the final PE=0 and the final KE to be (1/2) Iω2+ (1/2) ma2ω2 to finally obtain ω = √(3g/2a) But i would like to know...
  39. timthereaper

    Calculating Where to Push to Straighten a Rod

    I have some threaded rods on my 3D printer that I want to straighten. After searching Youtube for a quick easy method that doesn't involve a million "guess and check" steps, I found this: This guy seems to have concocted some method using Fourier series to straighten his rods. Not sure...
  40. Vivek98phyboy

    What are the other forces acting on ##dm## in addition to gravity?

    After solving using energy conservation, I found the angular velocity at 37° to be omega=2.97/(L)^½ Tension and the weight (dm)g are the two forces acting on the tip dm To find the resultant force, I resolved the centripetal force and tangential force to find the centripetal force as F=...
  41. Nexus99

    Moment of inertia, center of mass and vincular reaction of a rod

    I'm struggling doing point 5, i have no idea how to solve that question. In point 1 i obtained the following result: ## I=\frac{ML^2}{2}## calculating the integral of dI, the infinitesimal moment of inertia of a small section of the rod of length dr. 2) Through the conservation of angular...
  42. LCSphysicist

    Solving Waves on a Metal Rod with kx + Φ

    I am trying to solve this question by ξ = A*cos(ωt + θ)*sin(kx + Φ) Anyway, the two initial terms of the product helps nothing (i think), what matters is sin(kx + Φ) So, i tried by two ways: First: The stress is essentially zero on the ends, that is, something like cte*∂ξ/∂x (strain) would be...
  43. Nexus99

    What is the correct expression for the total moment of inertia in this scenario?

    1) I found: ##x_{CM} = z_{CM} = 0 ## ##y_{CM} = \frac{L}{2} \frac{m_1 - m_2}{M + m_1 + m_2} ## 2) Applying the conservation of linear momentum: ##M v_0 - m_1 v_0 - m_2 v_0 = (M + m_1 + m_2) v_f ## ##v_f = \frac{M - m_1 - m_2 }{M + m_1 + m_2} v_0 ## The velocity should have been found also...
  44. cwill53

    How much ice is needed to cool 31055.04J of water from 42°C to 0°C?

    My answer: a. $$H=\frac{dQ}{dt}$$ $$H_{brass}=H_{copper}$$ $$(109\frac{W}{m\cdot K})(0.005m^2)\frac{100^{\circ}C-T_{c}}{0.3m}=(385\frac{W}{m\cdot K})(0.005m^2)\frac{T_{H}-0^{\circ}C}{0.8m}$$ $$T_{H,Cu}=T_{C,brass}=T_{2}$$ $$-1.8166667T_{2}+181.666667^{\circ}C=2.40625T_{2}\Rightarrow...
  45. Nexus99

    A falling rotating rod strikes a ball of mass M....

    A homogeneous rod of length l and mass m is free to rotate in a vertical plane around a point A, the constraint is without friction. Initially the rod is stopped in the position of unstable equilibrium, therefore it begins to fall rotating around A and hits, after a rotation of ## \pi ## , a...
  46. Nexus99

    Rod on a Plane: Calculating Angular and Linear Motion After Impact"

    A hollow rod closed at the ends A and B, has mass M and length R. The rod is free to rotate on a horizontal frictionless plane around the z axis passing through A and coming out of the sheet. A body can slide without friction inside the cavity point mass m. Initially the rod is stationary and...
  47. C

    Finding the angular speed of a hinged rod without using torque or acceleration.

    The centre of mass of the rod would be at the middle of the rod i.e. at l/2=[50*10^(-2)]/2 The force responsible for torque will be acting downwards = mg The Torque = mg*l/2*sin(30) =mg*l/4 We know that Torque=I*alpha Hence alpha = mg*l/(4*I) Moment of inertia of rod about the end= ml^2/12 +...
Back
Top